K
Khách

Hãy nhập câu hỏi của bạn vào đây, nếu là tài khoản VIP, bạn sẽ được ưu tiên trả lời.

5 tháng 11 2016

a+b+c=3/2 => (a+b+c)2 = 9/4 <=> a2+b2+c2+2ac+2bc+2ac =9/4

mà ta có a2+b2+c2>= ac+bc+ac ( dễ dàng chứng minh được khi nhân hai lên rồi nhóm thành hằng đẳng thức hai số)

=> 3(a2+b2+c2)>= 9/4 <=> 4(a2+b2+c2) >= 4

=> min M=4 dấu bằng xảy ra <=> a=b=c=1/2

4 tháng 5 2017

mình nghĩ bạn Hoài có cách làm đúng nhưng kết quả sai

Mình dựa trên bài bạn thì được kết quả là Min=3 cơ

5 tháng 11 2016

Áp dụng BĐT BCS : \(\frac{3M}{4}=\left(1^2+1^2+1^2\right).\left(a^2+b^2+c^2\right)\ge\left(a+b+c\right)^2=\frac{9}{4}\Rightarrow M\ge3\)

Đẳng thức xảy ra khi a = b = c = 1/2

Vậy ..................................

NV
27 tháng 12 2020

\(M\ge\dfrac{\sqrt{\left(\sqrt{a}+\sqrt{b}\right)^2}}{2}+\dfrac{\sqrt{\left(\sqrt{b}+\sqrt{c}\right)^2}}{2}+\dfrac{\sqrt{\left(\sqrt{c}+\sqrt{a}\right)^2}}{2}\)

\(M\ge\sqrt{a}+\sqrt{b}+\sqrt{c}=3\)

Dấu "=" xảy ra khi \(a=b=c=1\)

10 tháng 11 2016

Ta có

\(2\left(a^2+b^2+c^2\right)\ge2\left(ab+bc+ca\right)\)

\(\Leftrightarrow3\left(a^2+b^2+c^2\right)\ge2\left(ab+bc+ca\right)+a^2+b^2+c^2=\left(a+b+c\right)^2\)

\(\Leftrightarrow4\left(a^2+b^2+c^2\right)\ge\frac{4}{3}.\left(a+b+c\right)^2=\frac{4}{3}.\frac{9}{16}=\frac{3}{4}\)

Đạt được khi \(a=b=c=\frac{1}{4}\)

10 tháng 11 2016

cửa hàng bán đc 640kg nhá

7 tháng 1 2020

4/ Xét hiệu: \(P-2\left(ab+7bc+ca\right)\)

\(=5a^2+11b^2+5c^2-2\left(ab+7bc+ca\right)\)

\(=\frac{\left(5a-b-c\right)^2+6\left(3b-2c\right)^2}{5}\ge0\)

Vì vậy: \(P\ge2\left(ab+7bc+ca\right)=2.188=376\)

Đẳng thức xảy ra khi ...(anh giải nốt ạ)

7 tháng 1 2020

@Cool Kid:

Bài 5: Bản chất của bài này là tìm k (nhỏ nhất hay lớn nhất gì đó, mình nhớ không rõ nhưng đại khái là chọn k) sao cho: \(5a^2+11b^2+5c^2\ge k\left(ab+7bc+ca\right)\)

Rồi đó, chuyển vế, viết lại dưới dạng tam thức bậc 2 biến a, b, c gì cũng được rồi tự làm đi:)

1 tháng 1 2016

Làm được bài này chưa. @@@

1 tháng 1 2016

Chưa, cậu làm được chưa?

14 tháng 1 2021

Theo nguyên lí Dirichlet, trong ba số a2, b2, c2 tồn tại 2 số cùng phía với 1.

Giả sử hai số đó là a2 và b2.

Ta có \(\left(a^2-1\right)\left(b^2-1\right)\ge0\Leftrightarrow\left(a^2+2\right)\left(b^2+2\right)\ge3\left(a^2+b^2+1\right)\)

\(\Rightarrow\left(a^2+2\right)\left(b^2+2\right)\left(c^2+2\right)\ge3\left(a^2+b^2+1\right)\left(1+1+c^2\right)\ge3\left(a+b+c\right)^2\) (Theo bất đẳng thức Cauchy - Schwarz).

Mà a + b + c = 3 nên \(S\ge27\).

Đẳng thức xảy ra khi a = b = c = 1.

Vậy Min S = 27 khi a = b = c = 1.